Peskin and Schroeder - Derivation of equation (2.45)

In summary: If so, then you should be able to use the first commutator to evaluate the second one and get the desired result.In summary, the conversation discusses a difficulty in deriving a particular equation and presents a derived expression. The summary goes on to show the steps taken in solving the equation and suggests evaluating the second commutator by breaking it up and using the first commutator to get the desired result.
  • #1
spaghetti3451
1,344
33
I'm having trouble deriving equation (2.45) on page 25. In particular, in the derivation of

##i\frac{\partial}{\partial t}\pi({\bf{x}},t) = -i(-\nabla^{2}+m^{2}) \phi({\bf{x}},t)##,

I need to show that

##\frac{1}{2}\pi({\bf{x}},t) \phi({\bf{x'}},t)(-\nabla^{2}+m^{2}) \phi({\bf{x'}},t) - \frac{1}{2} \phi({\bf{x'}},t)(-\nabla^{2}+m^{2})\phi({\bf{x'}},t)\pi({\bf{x}},t) = -i \delta^{(3)}({\bf{x}}-{\bf{x'}})(-\nabla^{2}+m^{2})\phi({\bf{x'}},t)##.

Now, this is what I've done so far:

##\frac{1}{2}\pi({\bf{x}},t) \phi({\bf{x'}},t)(-\nabla^{2}+m^{2}) \phi({\bf{x'}},t) - \frac{1}{2} \phi({\bf{x'}},t)(-\nabla^{2}+m^{2})\phi({\bf{x'}},t)\pi({\bf{x}},t)##

##=\frac{1}{2}\pi({\bf{x}},t) \phi({\bf{x'}},t)(-\nabla^{2}+m^{2}) \phi({\bf{x'}},t)-\frac{1}{2}\phi({\bf{x'}},t)\pi({\bf{x}},t) (-\nabla^{2}+m^{2}) \phi({\bf{x'}},t) +\frac{1}{2}\phi({\bf{x'}},t)\pi({\bf{x}},t) (-\nabla^{2}+m^{2}) \phi({\bf{x'}},t)- \frac{1}{2} \phi({\bf{x'}},t)(-\nabla^{2}+m^{2})\phi({\bf{x'}},t)\pi({\bf{x}},t)##

##=\frac{1}{2}[\pi({{\bf{x}},t}), \phi({{\bf{x'}},t})](-\nabla^{2}+m^{2}) \phi({\bf{x'}},t) + \frac{1}{2}\phi({\bf{x'}},t)[\pi({\bf{x}},t), (-\nabla^{2}+m^{2}) \phi({\bf{x'}},t)]##

##=\frac{1}{2}(-i)\delta^{(3)}({\bf{x'}}-{\bf{x}})(-\nabla^{2}+m^{2}) \phi({\bf{x'}},t) + ??##.

How do I evaluate the second commutator?
 
Physics news on Phys.org
  • #2
bumpp!
 
  • #3
Did you try breaking the second commutator up as [itex][\pi,m^2\phi] - [\pi,\nabla^2\phi][/itex] and convincing yourself that both the [itex]m^2[/itex] and [itex]\nabla^2[/itex] can be pulled out of their respective commutators?
 

Related to Peskin and Schroeder - Derivation of equation (2.45)

What is the derivation of equation (2.45) in Peskin and Schroeder?

The derivation of equation (2.45) in Peskin and Schroeder is based on the path integral formulation of quantum field theory. It involves calculating the propagator for a free quantum field and then using Feynman diagrams to calculate the scattering amplitude.

Why is equation (2.45) important in quantum field theory?

Equation (2.45) is important because it allows us to calculate the amplitude for a particle to propagate from one point to another in spacetime. This is a fundamental concept in quantum field theory and is used in many calculations, such as scattering processes and quantum corrections.

What are the assumptions made in the derivation of equation (2.45)?

The derivation of equation (2.45) assumes that we are dealing with a free quantum field, meaning there are no interactions between particles. It also assumes that the field is in its ground state, or vacuum state, and that it follows the principles of quantum mechanics.

How does equation (2.45) relate to Feynman diagrams?

Equation (2.45) is directly related to Feynman diagrams, as it is used to calculate the amplitude for a specific Feynman diagram. The calculation involves summing over all possible intermediate states and integrating over all possible momenta, resulting in the expression given by equation (2.45).

Can equation (2.45) be applied to interacting quantum fields?

No, equation (2.45) is only applicable to free quantum fields. When dealing with interacting quantum fields, more advanced techniques, such as perturbation theory, must be used to calculate the scattering amplitude.

Similar threads

Replies
41
Views
4K
  • Quantum Physics
Replies
9
Views
877
Replies
5
Views
1K
Replies
3
Views
862
Replies
5
Views
1K
Replies
3
Views
513
  • Quantum Physics
Replies
4
Views
999
Replies
5
Views
534
Replies
3
Views
2K
  • High Energy, Nuclear, Particle Physics
Replies
5
Views
1K
Back
Top